NURS 6550 FINAL EXAM / NURS6550 FINAL EXAM

NURS 6550 Final Exam / NURS6550 Final Exam (Latest): Walden University

Walden University NURS 6550 Final Exam / Walden University NURS6550 Final Exam

QUESTION 1

1. Mr. Jeffers was admitted 2 days ago for a carotid endarterectomy. A Foley catheter was inserted intraoperatively and remains in place. His urine output has declined markedly despite continued IV fluid infusion. Today his morning labs reveal a BUN of 19 mg/dL and a creatinine of 2 mg/dL. A leading differential includes:

 

A.Foley lodged in the urethra causing post-renal   failure

B.Decreased renal perfusion causing prerenal   failure

C.Age-related decreased eGFR causing prerenal   failure

D.Post-surgical rhabdomyolysis causing intrarenal   failure

QUESTION 2

1. Janet is admitted with symptomatic tachycardia. Her pulse is 160 b.p.m. and she is weak, diaphoretic, and anxious. Physical examination reveals a 5’4” 107 lb black female who is awake, alert, and oriented, anxious, with moist skin and racing pulse. Her blood pressure is 140/100 mm Hg. Temperature and respiratory rate are within normal limits. The patient admits to having a “thyroid condition” but she never followed up on it when she was advised to see an endocrinologist. The AGACNP anticipates a diagnosis of:

 

A.

Hashimoto’s thyroiditis

 

B.

Cushing’s syndrome

 

C.

Grave’s disease

 

D.

Addison’s disease

QUESTION 3

1. Systemic lupus erythematosis (SLE) is a multiorgansystem autoimmune disorder that can prevent with a wide variety of manifestations. Which clinical triad should prompt an evaluation for SLE?

 

A.

Fever, normal white count, elevated sedimentation   rate

 

B.

Hyperkalemia, hyponatremia, low blood pressure

 

C.

Leukocytosis, hyperglycemia, hypokalemia

 

D.

Joint pain, rash, fever

QUESTION 4

1. A patient presents with profound vertigo of acute onset yesterday. She can barely turn her head without becoming very vertiginous; she is nauseous and just doesn’t want to move. This morning when she tried to get out of bed she felt like she was pushed back down. The vertigo is reproducible with cervical rotation. The patient denies any hearing loss or tinnitus, she has no fever or other symptoms. The AGACNP knows that the most helpful intervention will probably be:

 

A.

Meclizine

 

B.

Diazepam

 

C.

Bed rest

 

D.

Epley’s maneuvers

QUESTION 5

1. Mrs. Mireya is an 85-year-old female who is admitted for evaluation of acute mental status change from the long term care facility. She is normally ambulatory and participates in lots of facility activities. Today a nursing assistant found her in her room, appearing confused and disconnected from her environment. When she tried to get up she fell down. Her vital signs are stable excepting a blood pressure of 90/60 mm Hg. The AGACNP knows that the most likely cause of her symptoms is:

 

A.

Osteoarthritis

 

B.

Drug or alcohol toxicity

 

C.

Hypotension

 

D.

Urosepsis

QUESTION 6

1. A patient with SIADH would be expected to demonstrate which pattern of laboratory abnormalities?

 

A.

Serum Na+ 119 mEq/L, serum osmolality 240 mEq/L,   urine Na+ of 28 mEq/L, urine osmolality of 900 mOsm/kg

 

B.

Serum Na+ 152 mEq/L, serum osmolality 315 mEq/L,   urine Na+ of 5 mEq/L, urine osmolality of 300 mOsm/kg

 

C.

Serum Na+ 121 mEq/L, serum osmolality 290 mEq/L,   urine Na+ of 7 mEq/L, urine osmolality of 850 mOsm/kg

 

D.

Serum Na+ 158 mEq/L, serum osmolality 251 mEq/L,   urine Na+ of 20 mEq/L, urine osmolality of 420 mOsm/kg

QUESTION 7

1. Sean is a 29-year-old male who presents to the emergency department for evaluation and treatment of foreign body in the eye. Ophthalmic anesthesia is achieved and removal is attempted unsuccessfully with a moist cotton tipped swab. A wet fluorescein stain is applied to the lower eyelid, and a corneal abrasion ruled out but the AGACNP notes a positive Seidel sign. This indicates:

 

A.

Penetration of the cornea with resultant aqueous   leak

 

B.

A rust ring remnant due to metal foreign body

 

C.

An elevated intraocular pressure

 

D.

Paradoxical pupil dilation in response to light

QUESTION 8

1. Mrs. Lowen is an 82-year-old female who comes to the emergency department for evaluation of a fever of 102.9° F. She complains of a headache in the right side of her temple and some right-sided jaw pain. A urinalysis, chest radiograph, complete blood count (CBC) and 12-lead ECG are all non-contributory. A comprehensive metabolic panel is significant only for a slightly elevated BUN and creatinine. The AGACNP appreciates distinct right temple tenderness to percussion. Which laboratory test is necessary to support the suspected diagnosis?

 

A.

An erythrocyte sedimentation rate

 

B.

A white blood cell differential

 

C.

Two sets of blood cultures

 

D.

Echocardiography

QUESTION 9

1. Ms. Schiebel, a 31-year-old female who is brought to the emergency department by police after being arrested for disruptive behavior in a public establishment. The differential diagnosis includes drug and alcohol ingestion/toxicity, central nervous system disease, severe trauma, and psychotic illness; ultimately the alcohol and toxicology screen as well as head imaging are negative. When considering psychotic illness, the AGACP knows that this is a physiologic imbalance that typically involves an excess of:

 

A.

Serotonin

 

B.

Norepinephrine

 

C.

Acetylcholine

 

D.

Dopamine

QUESTION 10

1. Mr. Lincoln is a 55-year-old male who was admitted for management of sepsis secondary to pneumonia. He has declined rapidly, and today chest radiography demonstrates a diffuse, bilateral “white-out” appearance. His paO2 is 55 mm Hg. In order to increase his oxygenation the AGACNP knows that which of the following interventions is indicated?

 

A.

Increased   FiO2

 

B.

Increased respiratory rate

 

C.

Increased   tidal volume

 

D.

Increased PEEP

QUESTION 11

1. A 29-year-old female patient presents with a complaint of palpitations. Physical examination reveals an essentially healthy female with no significant medical history and no maintenance medications; the only thing she can report is that she had a head cold a week or so ago. The vital signs include a blood pressure of 139/90 mm Hg, pulse of 105 b.p.m, respiratory rate of 16 b.p.m. and a temperature of 98.6° F. The only abnormal finding on physical examination is diffuse anterior neck tenderness with thyroid palpation. The AGACNP considers which medication for symptom control?

 

A.

Ibuprofen

 

B.

Pseudoephedrine

 

C.

Propranolol

 

D.

Methimazole

QUESTION 12

1. Jennifer is an 18-year-old homeless female who was found unresponsive. She was admitted to the hospital for management of severe bleeding after a spontaneous abortion escalated to a uterine hemorrhage. An underlying infection and dehydration were corrected and nutritional supplements were started. Her volume status is stable, morning labs were all within normal limits and she is to be discharged today. When the AGACNP enters the room to prepare the patient for discharge, she finds her agitated, pale, and diaphoretic with vital signs to include a pulse of 105 bpm, respirations of 24 bpm, blood pressure of 110/76 mm Hg and a temperature is 97.9° F. The most appropriate action would be to:

 

A.

Order a CBC to assess for recurrent bleeding

 

B.

Request and abdominal CT to assess for bleeding

 

C.

Evaluate the patient for anxiety/panic attack

 

D.

Prescribe alprazolam 1 mg now

QUESTION 13

1. Physical examination findings in a patient with pneumothorax is likely to reveal:

 

A.

Increased tactile fremitus

 

B.

Low grade temperature

 

C.

Hyperresonance to percussion

 

D.

Egophany

QUESTION 14

1. Mr. Parker brings his 73-year-old wife to a clinic appointment because he is worried about her. She has a long history of hypertension and dyslipidemia, but he says she has taken medication for years and everything has been OK. His concern today is that for a long time she has been very forgetful, and he has tried to help her by keeping a strict routine around the house. Over the past few months, she just seems more and more forgetful, does not seem interested in doing anything, and now seems to be forgetting how to do simple everyday tasks. Yesterday she could not figure out which dollar bills to use at the store to pay the cashier. The AGACNP knows Mrs. Parker should first be screened for:

 

A.

Depression

 

B.

A brain tumor

 

C.

Hypothyroidism

 

D.

Adrenal dysfunction

QUESTION 15

1. M.R. is a 40-year-old female who has a known history of peptic ulcer disease. She has been admitted through the emergency room with a diagnosis of GI bleeding—she is vomiting dark blood and had a nasogastric tube placed. When attached to low intermittent suction it initially drained 400 cc of dark brown/black drainage, but now it is starting to drain lighter red colored blood. The AGACNP knows that immediate priorities of care include:

 

A.

Ensuring hemodynamic stability

 

B.

Beginning a parenteral proton pump inhibitor

 

C.

Beginning gastric lavage

 

D.

Ordering a gastrointestinal consult

QUESTION 16

1. A patient with sharp, stabbing chest pain directly over the precordium has a 12-lead ECG that demonstrates concave ST-T wave elevations in leads II, III, avR, avL, avF, and all six precordial leads. The AGACNP expects which physical finding?

 

A.

A grade IV/VI systolic murmur with radiation to   the axilla

 

B.

A split S2 that increases with inspiration

 

C.

A pericardial friction rub

 

D.

An S4 heart sound

QUESTION 17

1. J.Q. is a 45-year-old male who had gastric bypass surgery 18 months ago. A CBC reveals a macrocytic anemia with aHgb of 9.8 g/dL, HCT of 30%, MCV of 115 and RDW of 19%. The AGACNP suspects which type of anemia?

 

A.

Iron deficiency

 

B.

Sickle cell anemia

 

C.

Pernicious anemia

 

D.

Anemia of chronic disease

QUESTION 18

1. Megan K. is a 21-year-old female who presents complaining of irritated eyes. She says this happens a couple of times a year and this time it is really a problem. Both eyes are itchy and red and she has a lot of stringy discharge, especially at the end of the day. Her visual acuity is 20/25 OS, OD, and OU with her glasses on. Physical exam reveals injected conjunctiva bilaterally but there is no photophobia. Pupils are equal, round, briskly reactive, and accommodate. The AGACNP knows that immediate treatment should include ophthalmic application of:

 

A.

Steroids

 

B.

Antihistamine

 

C.

Antibiotic

 

D.

Cycloplegic

QUESTION 19

1. Ellen is a 61-year-old female who presents with a chief complaint of neck pain. The history of present illness reveals that Ellen felt as though a bug bit her behind the neck a few days ago. A day or two later it started to hurt, and when she began to pick at it she felt drainage come out. She is here now for evaluation. Physical exam reveals an 8 cm x 8 cm draining abscess in the right post auricular region with posterior cervical lymphadenopathy. Ellen has a temperature today of 101.9° F. The AGACNP knows that in addition to incision and drainage of the abscess, effective management must include:

 

A.

Systemic antibiotics

 

B.

Tetanus immune globulin

 

C.

Tetanus toxoid

 

D.

Antipyretics

QUESTION 20

1. A 13-year-old male presents with a chief complaint of ear drainage. The patient and his mother both indicate that the patient has not had any pain or any systemic complaints, but the pus-like discharge from the ear is very persistent. According to Mom they went to a retail clinic two weeks ago and the patient was prescribed both oral antibiotics and ear drops, but it didn’t help. Physical exam of the ear reveals a painless pinna; otoscope exam reveals only a large amount of mucopurulent drainage—the tympanic membrane could not be visualized. The AGACNP knows the diagnosis is most likely:

 

A.

Acute otitis media

 

B.

Acute otitis externa

 

C.

Cholesteatoma

 

D.

Otitis media with effusion

QUESTION 21

1. A 71-year-old male patient with lung cancer is admitted for treatment of sepsis related to his chemotherapy-induced immunosuppression. He seems to be improving from an infectious perspective, but during today’s assessment the AGACNP appreciates coarse rales in the lung fields, a blood pressure of 140-100 mm Hg, a bounding pulse, and trace pretibial edema. The urine output via Foley catheter has only been 100 mL in the last 8 hours. Suspicious for syndrome of inappropriate antidiuretic hormone (SIADH), the AGACNP orders a basic metabolic panel anticipating which of the following abnormalities?

 

A.

Hypokalemia

 

B.

Hypocalcemia

 

C.

Hyponatremia

 

D.

Hypochloremia

QUESTION 22

1. A crescendo-decrescendo systolic murmur best appreciated at the second intercostal space, right sternal border with radiation to the carotid artery is most likely an indicator of:

 

A.

Aortic stenosis

 

B.

Aortic regurgitation

 

C.

Tricuspid stenosis

 

D.

Tricuspid regurgitation

QUESTION 23

1. The AGACNP knows that diagnostic findings consistent with rheumatoid arthritis include:

 

A.

Soft tissue swelling of the metacarpals

 

B.

Radiographic joint space narrowing

 

C.

Heberden’s nodes

 

D.

Subungal hemorrhages

QUESTION 24

1. C.T. is a 39-year-old female who presents for evaluation of what she thinks is her “rosacea acting up.” She has a history of acne rosacea and has medicated on and off for years with tetracycline and topical metronidazole. Today however she presents with a pronounced red/purple area on her left cheek extending to the nasal border. It is very warm to the touch. The borders of the affected area are very well defined and raised. C.T. also has a temperature of 100.7° F and a generalized headache. The AGACNP appreciates tender submandibular and cervical lymphadenopathy. The likely diagnosis is:

 

A.

Complex rosacea

 

B.

Cellulitis

 

C.

Erysipelas

 

D.

Allergic reaction

QUESTION 25

1. Mr. Lopez is a 51-year-old male patient who is being treated for T2DM. His HgbA1c is 15.6% and initial management will include aggressive attempts for weight reduction as his body mass index (BMI) is 45. He says he is unable to participate in any meaningful exercise because he very often has back pain; he has had it for years and has tried all sort of over the counter medicines with little relief. He describes it as a profound ache that occurs across the lower part of his back bilaterally; it does not travel down either leg. The physical inspection is normal, but he has significant paraspinal tenderness to palpation bilaterally. He cannot identify any injury or accident that preceded the pain. The history and physical exam is noncontributory. The AGACP knows that the likely diagnosis is:

 

A.

Lumbar radiculopathy

 

B.

Ankylosing spondylitis

 

C.

Lumbar sacral strain

 

D.

Degenerative disk disease

QUESTION 26

1. A patient presents with acute onset of vesicular lesions on her vulva. They are surrounded by areas of redness and they hurt. The patient says that she has even more of them now then she did when she woke up this morning. There is also inguinal lymphadenopathy. The AGACNP is suspicious for:

 

A.

Human papilloma virus

 

B.

Primary syphilis

 

C.

Gonorrhea

 

D.

Herpes simplex virus

QUESTION 27

1. Classic radiographic features of osteoarthritis include:

 

A.

Soft tissue swelling

 

B.

Joint deformity

 

C.

Bone mineral loss

 

D.

Joint space narrowing

QUESTION 28

1. Mrs. Sandoval is a 72-year-old female who presents with a chief complaint of transient verbal confusion. She was speaking with her friend on the phone this morning when she suddenly couldn’t get words out. Her friend went over to her home and found Mrs. Sandoval awake, alert, and oriented, responding appropriately with non-verbal gestures, but she could not properly articulate her thoughts. By the time she arrived at the office this had passed, although during the examination she appeared to have infrequent difficulty finding a single word. The patient denies any contributory medical history, but a 12-lead ECG in the office reveals atrial fibrillation with a ventricular response of 91 b.p.m. The blood pressure is 140/94 mm Hg; remaining vital signs are normal. The AGACNP knows that management should include:

 

A.

Antiplatelet therapy

 

B.

Anticoagulation

 

C.

Blood pressure control

 

D.

Speech therapy

QUESTION 29

1. C.L. is a 48-year-old female who presents complaining of activity intolerance. She is usually very active and fit^. She jogs regularly and typically does 4-5 miles a day. About a week ago she became so tired she had to stop, and lately she has become aware of becoming easily fatigued while going up and down stairs. She admits that she thinks she is beginning menopause—she is having a lot of bleeding with her periods, and her periods seem to be more frequent. A complete blood count (CBC) reveals the following results:
Hgb 10.1 g/dL
Hct 30%
MCV 75 fL
RDW 21%
The AGACNP orders which of the following laboratory test to confirm the suspected diagnosis?

 

A.

Vitamin B12

 

B.

Folate

 

C.

Ferritin

 

D.

Hemoglobin electrophoresis

QUESTION 30

1. Kevin H. is a 61-year-old male who presents for treatment of profound anxiety. He has been treated on and off for years—most recently he was taking escitalopram 20 mg p.o. daily, and although he does admit to some improvement, he still cannot function appropriately thoughout the day. He has been counseled about poor work performance and is concerned about losing his job, but he is just so worried all of the time he cannot concentrate on work. The AGACNP knows that the most appropriate action is to:

 

A.

Increase the dose of escitalopram to 40 mg daily

 

B.

Refer Kevin for a psychiatric consultation

 

C.

Stop escitalopram and begin venlafaxine

 

D.

Discuss therapeutic expectations with Kevin

QUESTION 31

1. When examining a patient with a skin presentation suggestive of necrotizing fasciitis, the AGACNP knows that the most important and sensitive diagnostic test is:

 

A.

A complete blood count

 

B.

Plain film radiographs

 

C.

The finger test

 

D.

CT scan

QUESTION 32

1. While evaluating a patient with abdominal pain, the AGACP knows that when the pain is described as coming in waves or cycles, with periods of relief in between, the cause likely centers around:

 

A.

Peristalsis of bowel

 

B.

Disorders of pelvic organs

 

C.

Organ inflammation

 

D.

Hyperacidity

QUESTION 33

1. Which of the following findings is not typically associated with testicular torsion?

 

A.

Acute pain

 

B.

Edema

 

C.

High riding testis

 

D.

Dysuria

QUESTION 34

1. 152: When completing this exam, did you comply with Walden University’s Code of Conduct including the expectations for academic integrity?

 

Yes

 

No

QUESTION 35

1. While preparing to perform an incision and drainage on a 7 cm fluctuant abscess on a patients posterior thorax, the AGACNP knows that the most important part of the procedure is:

 

A.

Immediate coverage with antistaphylococcal   antibiotics

 

B.

Maintaining sterility with topical betadine and   drapes

 

C.

Breaking up loculations and aggressive irrigation

 

D.

Proper injection of local anesthetic

QUESTION 36

1. A patient is being evaluated with significant nausea, fatigue, and a general sense of feeling unwell; mild jaundice is noted on physical examination. Transaminases are markedly elevated and a hepatitis screening is done. Results are as follows:

+ HbsAb

+ anti-HAV IgM

– anti-HCV

The correct interpretation of these findings is:

 

A.

The patient has acute hepatitis A

 

B.

The patient has acute hepatitis B

 

C.

The patient has chronic hepatitis B

 

D.

The patient has acute hepatitis C

QUESTION 37

1. When treating a patient with an unknown overdose or toxicity, the AGACNP knows that all of the following should be administered except:

 

A.

Dextrose 50%

 

B.

Thiamine 100 mg

 

C.

Nalaxone 0.4 mg

 

D.

Ativan   4 mg

QUESTION 38

1. The AGACNP is evaluating 29-year-old female who presents by ambulance and is unresponsive. There is no witness and no history available; the patient is not wearing any sort of medic alert bracelet. While assessing for toxicity or overdose, the patient is found to have vital signs as follows: Temp of 96.2° F, pulse of 48 b.p.m., respirations of 10 b.p.m., and blood pressure of 84/50 mm Hg. The patient’s pupils are constricted, but do react briskly to light to 1 mm. The AGACNP suspects which type of substance?

 

A.

Cholinesterase inhibiting drugs

 

B.

Stimulants such as MDMA

 

C.

Anticholinergics

 

D.

Ethanol or opiates

QUESTION 39

1. The AGACNP knows that the one class of pain medication that is effective to some extent for all forms of pain is:

 

A.

NSAIDs

 

B.

Antidepressants

 

C.

Antiepileptics

 

D.

Opiates

QUESTION 40

1. K.P. is a 76-year-old male admitted for antibiotic management of urosepsis. His medical history is significant for a CVA with resultant right-sided hemiparesis. He is nonverbal, maintained on enteral nutritional support and has an indwelling Foley catheter. The AGACNP knows that which of the following bacteria is the primary treatment target for this patient’s urosepsis?

 

A.

Proteus   mirabilis

 

B.

Pseudomonas aeruginosa

 

C.

Staphylococcus aureus

 

D.

Streptococcus pneumoniae

QUESTION 41

1. A patient is admitted for a COPD exacerbation and placed on mechanical ventilation. His settings are as follows: FiO2 of 40%, TV of 700mL, SIMV of 12. His morning ABG reveals a pH of 7.37, paCO2 of 51 mm Hg, paO2 of 84 mm Hg and HCO3 of 30 mm Hg. The AGACNP knows that the appropriate response is to:

 

A.

Leave the ventilator settings as is

 

B.

Increase the SIMV to 16 b.p.m.

 

C.

Increase   the FiO2 to 50%

 

D.

Repeat the ABG in one hour

QUESTION 42

1. All of the following are required for a diagnosis of systemic inflammatory response syndrome (SIRS) except:

 

A.

White blood cell count < 4000 or > 12,000   cells/uL

 

B.

Heart rate > 90 b.p.m.

 

C.

Respiratory   rate > 20 b.p.m. or paCO2 < 32 mm Hg

 

D.

Two   sets of positive blood cultures

QUESTION 43

1. J.T. is a 41-year-old female patient who presents with a chief complaint of “heartburn.” She says that it doesn’t really seem to be related to meals or food—it occurs at random times. She does note, when asked, that it seems to happen a lot at night and occasionally wakes her up. Her only other symptom complaint is an occasional cough. It does not produce mucus, and she admits to assuming it was a “nervous” cough. The next appropriate action for the AGACNP would be to:

 

A.

Order an H. pylori test

 

B.

Request   a GI consult for endoscopy

 

C.

Order a proton pump inhibitor 30 minutes before   breakfast

 

D.

Request a 72-hour diet history

QUESTION 44

1. Your patient has diabetes insipidus (DI). Anticipated physical assessment findings include:

 

A.

Dry skin, tachycardia, hypertension

 

B.

Weak pulse, dry skin, decreased skin turgor

 

C.

Thin hair, thready pulse, dry mucous membranes

 

D.

Hypothermia, jugular venous distention,   bradycardia

QUESTION 45

1. The AGACNP is beginning medical management of a patient newly diagnosed with T2DM. The patient has a BMI of 39 and has been unsuccessful in making significant diet and lifestyle changes over the last six months. Other than her weight, her physical examination is essentially within normal limits. Her HgbA1c is 9.5%. A basic metabolic panel is within normal limits. The medication of choice to begin therapy will be:

 

A.

A sulfonyurea

 

B.

A meglitinide

 

C.

A biguanide

 

D.

An incretin mimetic

QUESTION 46

1. Felty’s syndrome is a condition of immune neutropenia seen sometimes in patients with:

 

A.

Polymyalgia rheumatica

 

B.

Giant cell arteritis

 

C.

Systemic lupus erythematosus

 

D.

Rheumatoid arthritis

QUESTION 47

1. When treating a patient for the profound cough of acute bronchitis, the AGACNP knows that the most appropriate pharmacotherapy consists of:

 

A.

An opiate based cough suppressant

 

B.

Oral prednisone

 

C.

A first generation-antihistamine combination

 

D.

An inhaled anticholinergic

QUESTION 48

1. Mr. Truman is transferred to the emergency department by ambulance. His wife called 911 this morning because he was acting “funny” when he woke up. Both the patient and his wife went to bed last night at approximately 10:30 and everything was normal. This morning he could not communicate orally and seemed confused about how to ambulate. Upon arrival to the emergency department his vital signs are as follows: Temperature 100.9° F, pulse 89 b.p.m., respirations 14 b.p.m. and blood pressure 168/94 mm Hg. A non-contrast CT scan of the head reveals thrombotic CVA. The AGACNP know that immediate management of this patient should include:

 

A.

Thrombolytics

 

B.

IV vasodilators

 

C.

Aspirin

 

D.

Antiepileptics

QUESTION 49

1. Your patient is complaining of profound nausea and vomiting that started at bedtime last night and kept him awake all night long. Early this morning he started having abdominal cramping and explosive diarrhea. Based upon the character of symptoms you are suspicious of infection with Staphylococcus aureus. To assess risk for exposure to this organism, you ask the patient about which meal?

 

A.

Breakfast yesterday

 

B.

Lunch yesterday

 

C.

Dinner yesterday

 

D.

Bedtime snack yesterday

QUESTION 50

1. D.R. is a 54-year-old male patient who was admitted for the management of cellulitis and treated with parenteral antibiotics. He has not been responding as well as anticipated. During today’s exam the AGACNP appreciates a couple of changes. All of the following indicate the need for immediate surgical evaluation except:

 

A.

Skin anesthesia

 

B.

Violaceous bullae

 

C.

Gas bubbles in tissue

 

D.

Lymphangetic spread

QUESTION 51

1. R. O. is a 21-year-old female who comes to the emergency department because of a severe headache. Her vital signs and neurological examination are within normal limits. She complains of a pulse-like pain in her right temple and admits that she has almost vomited. Her mother gets the same type of headache and the last time this happened R.O. took one of her mother’s prescription headache pills. They helped a lot, but this time her mother told her she had to come be evaluated. The AGACNP knows that which of the following is the appropriate action?

 

A.

A non-contrast CT scan of the head

 

B.

Administration of a 5HT agonist

 

C.

Dilaudid 2 mg IM x 1 dose

 

D.

Requesting a headache diary

QUESTION 52

1. A 39-year-old female presents for evaluation of a rash. She denies any significant medical history, and has no other complaints. The rash appeared suddenly on both forearms approximately one week ago, and she is concerned because it is not going away. It does not itch or hurt—it is just there. Physical examination reveals a diffuse macular hypopigmentation on both forearms that extends to the hands. The patient denies any drug or alcohol use; she is single and has had 4 unprotected sexual partners in the last year. The AGACNP knows that initial laboratory testing must include a(n):

 

A.

FTA-Abs

 

B.

Fungal skin scraping

 

C.

RPR screening

 

D.

CBC

QUESTION 53

1. J.S. is a African-American female who presents for a wellness examination. Her medical history is significant for beta thalassemia minor. Anticipated red blood cell differential would include which of the following patterns?

 

A.

Hgb 10.2 g/dL, Hct 30%, MCV 70 fL, RDW 12.6%

 

B.

Hgb 9.9 g/dL, Hct 28%, MCV 83 fL, RDW 13.9%

 

C.

Hgb 11.5 g/dL, Hct 35%, MCV 94 fL, RDW 15.8%

 

D.

Hgb 12.8 g/dL, Hct 38%, MCV 105 fL, RDW 18.1%

QUESTION 54

1. M.T. presents complaining of acute pain in his left eye, nausea, and one episode of vomiting. He denies any significant medical problems, and says that the only medication that he takes is an occasional over-the-counter sleeping pill. Physical examination reveals a steamy red cornea and conjunctiva with a pupil that is 5 mm and not reactive to light. The AGACNP knows that diagnostic testing should include:

 

A.

A CT scan of the head

 

B.

An MRI of the orbit

 

C.

A toxicology screen

 

D.

A measurement of intraocular pressure

QUESTION 55

1. Justin is a 23-year-old male who is being managed for an acute manic episode. Justin was diagnosed with bipolar disorder several years ago, but his home life has been unstable and he has not been very adherent to a medication regimen. Most recently he was started on the SNRI venlafaxine by his primary care provider, which he has been taking as prescribed for about 6 weeks, but he began a manic episode a few days ago which peaked this evening. The AGACNP considers that:

 

A.

The manic episode is probably a result of   medication instability and he should continue his current regimen with a   follow-up in 6-8 weeks

 

B.

A mood stabilizing agent should be added to the   venlafaxine

 

C.

All medication should be held for 6-8 weeks and   the then the patient should be reevaluated

 

D.

The SNRI should be stopped and a mood stabilizing   agent started

QUESTION 56

1. Mr. Livingston is a 79-year-old male who presents from a long term care facility with a change in mental status. His medical history is significant for T2DM, CAD, CHF, hypothyroidism, Alzheimer’s dementia and osteoarthritis. He has been stable, but over the last few days the staff say he has been a bit disconnected. This morning he was found in his bed in a stuporous state. His vital signs include a temperture of 98.9° F, pulse of 103 b.p.m., respiratory rate of 20 b.p.m., and a blood pressure of 92/64 mm Hg. His metabolic panel demonstrates a Na+ of 129 mEq/L, K+ of 3.3 mEq/L, Cl- of 100 mEq/L, CO2 of 24 mEq/L, glucose of 644 mg/dL, BUN of 51 mg/dL and creatinine of 1.9 mg/dL. The AGACNP knows that the primary problem is most likely:

 

A.

Diabetic ketoacidosis

 

B.

Hypertonic hyponatremia

 

C.

Myxedema coma

 

D.

Hyperosmolar hyperglycemic coma

QUESTION 57

1. The AGACNP is evaluating a patient with systemic lupus erythematosis who complains of fatigue. Based upon his knowledge of the most commonly affected visceral organ, which of the following diagnostic studies should be ordered?

 

A.

Echocardiogram

 

B.

Chest radiography

 

C.

Hepatic function enzymes

 

D.

Urinalysis with microscopic

QUESTION 58

1. All of the following are true statements about post-traumatic stress disorders (PTSD) except:

 

A.

It is more common in women than men

 

B.

It is unlikely to occur in children especially   < 10 years old

 

C.

It is differentiated from acute stress reaction   by time

 

D.

It is not likely in persons with no preexisting   psychiatric disease

QUESTION 59

1. Ray M., a 49-year-old male, walks into the emergency room complaining of back pain. He has never had this problem before and cannot identify any injury, but he is in such severe pain he is sure something is wrong. He states that his back has been hurting so badly sometimes he has to stop whatever he is doing and bend forward at the waist. The pain also travels along the outer edge of his left thigh to mid-calf, and he reports a small area of numbness on his anterior thigh. His history and physical examination are otherwise negative. He is an insurance attorney and is not especially active at work, but goes to the gym 5 days a week. He is not overweight, and his vital signs are normal. Physical examination reveals no paraspinal tenderness, and his straight leg raise is negative. A few times during the exam he lay back on the table and grabbed his left leg, flexed both hip, and pulled his knee to his chest, because it helped the pain. The AGACNP knows that immediate pain relief measures must include:

 

A.

An opiate analgesic

 

B.

Systemic steroids

 

C.

Physical therapy

 

D.

Bedrest for 72 hours

QUESTION 60

1. A patient with peptic ulcer disease is admitted to the hospital with significant upper abdominal discomfort. She has guarding and rebound tenderness on examination. Abdominal radiography demonstrates free air in the abdomen. The AGACNP knows that the immediate priority is to:

 

A.

Obtain a stat surgical consult

 

B.

Begin an IV proton pump inhibitor

 

C.

Order an abdominal CT scan

 

D.

Obtain a stat gastroenterology consult

QUESTION 61

1. Jennifer is a 15-year-old female who attempted suicide by taking a bottle of acetaminophen. She took 30, 500 mg tablets approximately six hours ago, but then became frightened and told her mother what she did. Her mother said that Jennifer seems OK, other than being a little sick to her stomach, she has no complaints. The AGACNP knows that the first step in her care includes:

 

A.

N-acetycysteine in tapering doses over the next   24 hours

 

B.

Oral administration of activated charcoal

 

C.

Psychiatric assessment

 

D.

Discharge to home with follow-up LFTs in 4 days

QUESTION 62

1. Mrs. Glassman is a 55-year-old female who presents with a chief complaint of fever. Her vital signs reveal a temperature of 100.0° F, blood pressure of 100/60 mm Hg, pulse of 114 b.p.m. and respirations of 20 b.p.m. Her cardiac auscultation reveals a grade III/VI systolic murmur at the left lower sternal border. Her history is significant for an eyebrow lift 4 months ago. The AGACNP orders which test to confirm the suspected diagnosis?

 

A.

Three sets of blood cultures

 

B.

A chest radiograph

 

C.

A 12-lead ECG

 

D.

Induced sputum culture

QUESTION 63

1. John is a 17-year-old male who is in the emergency department with abdominal pain. He is quite uncomfortable and says that it started yesterday and seemed to be “in the middle of his stomach” but today it has moved over to the right lower side. During physical examination the abdomen is not distended, but he is guarded, and right lower quadrant palpation produces significant discomfort, especially upon release of the palpating hand. He has appreciable pain when his right knee and hip are bent to a 90° angle. John admits to some nausea but has not vomited; he has not had a normal bowel movement in two days. His vital signs are as follows: Temperature 100.9° F, pulse 110 b.p.m. respiratory rate 22 b.p.m., and blood pressure 118/77 mm Hg. The AGACNP orders which of the following tests to confirm the suspected diagnosis?

 

A.

Complete blood count

 

B.

Ultrasound

 

C.

CT scan

 

D.

Urinalysis

QUESTION 64

1. Which of the following signs is expected in patients with cholecystitis?

 

A.

McBurney’s

 

B.

Cullen’s

 

C.

Spurling’s

 

D.

Murphy’s

QUESTION 65

1. According to the World Health Organization’s step-wise approach to pain management, initial approaches to step 2 might include all of the following except:

 

A.

A weak opiate

 

B.

A strong opiate

 

C.

A non-steroidal antiinflammatory agent

 

D.

An antidepressant.

QUESTION 66

1. A patient’s Weber test lateralizes to the right ear and the Rinne test in both ears is normal. The patient has a:

 

A.

Sensorineural hearing loss in the left ear

 

B.

Sensorineural hearing loss in the right ear

 

C.

Conductive hearing loss in the left ear

 

D.

Conductive hearing loss in the right ear

QUESTION 67

1. J.B. is a 62-year-old male who was admitted three days ago for management of diverticulitis. Today the AGACNP is called to the bedside to evaluate new onset swelling of the right lower extremity. According to the staff nurse it was not present yesterday but on today’s assessment the patient had 2A+ edema up to the thigh. Initial diagnostic evaluation should include:

 

A.

Homan’s sign

 

B.

A venogram

 

C.

A D-dimer

 

D.

CT of the chest

QUESTION 68

1. Based upon clinical examination and laboratory assessment the AGACNP diagnoses a patient with giant cell arteritis. The next step in the patient management should be to:

 

A.

Consult surgery for a temporal artery biopsy

 

B.

Consult rheumatology for medical management

 

C.

Order 60 mg of prednisone now and q.d.

 

D.

Order ceftriaxone 1 mg IV now

QUESTION 69

1. According to the JNC VIII criteria, a patient with a new diagnosis of hypertension who has comorbid chronic kidney disease should be started on which of the following classes of medications?

 

A.

A thiazide diuretic

 

B.

A calcium channel blocker

 

C.

An ACE inhibitor

 

D.

A beta adrenergic antagonist

QUESTION 70

1. Denise is a 45-year-old female who presents with significant lower abdominal pain. It started a few days ago and has just gotten steadily worse. She denies any hematuria or dysuria, but when she voids she feels like “everything is coming out. A physical examination reveals an abdomen that is tender to palpation but there is no guarding or rebound. Her vital signs are stable excepting a temperature of 100.9° F. The next step in the evaluation must include:

 

A.

A complete blood count

 

B.

An abdominal flat plate

 

C.

A pelvic examination

 

D.

A CT scan of the abdomen

QUESTION 71

1. Jan is a 39-year-old female who presents with significant right upper quadrant pain of 18 hours duration. She admits to a few episodes of vomiting. She right upper quadrant pain to palpation but the ultrasound is negative. Jan admits that this has happened before, usually when she “eats a huge meal.” The AGACNP orders which diagnostic study to confirm the diagnosis of cholecystitis?

 

A.

Upright abdominal radiography

 

B.

Hepatic function panel

 

C.

HIDA scan

 

D.

Abdominal CT

QUESTION 72

1. L.W. is a 41-year-old woman with a history of systemic lupus erythematosus which has been managed primarily with symptom control. Today she presents for evaluation of fatigue which has been slowly progressive over the last few months. She has a history of gastric bypass surgery 10 years ago and has maintained a 100 lb weight loss, but she maintains that she has been very adherent to her vitamin and mineral replacement regimen. Other than chronically heavy menses, for which she takes hormonal contraception, she is without complaint. A complete blood count is as follows:
Hgb 10.3 g/dL
Hct 31%
MCV 88 fL
RDW 15%
The AGACNP suspects that the patient’s fatigue is most likely due to:

 

A.

Iron deficiency anemia

 

B.

Anemia of chronic disease

 

C.

Pernicious anemia

 

D.

Folic acid deficiency

QUESTION 73

1. A patient presents for follow up after being started on an ACE inhibitor for hypertension. Her blood pressure has improved, but her pulse is 56 b.p.m down from 76 b.p.m. at her last visit. The AGACNP knows that the patient should assessed for:

 

A.

Hypercalcemia

 

B.

Hypernatremia

 

C.

Hyperkalemia

 

D.

Hyperchloremia

QUESTION 74

1. A young-adult male patient was dropped off outside of the emergency department and some staff members brought him inside. The patient is restless, irritable, and either unwilling or unable to participate in her own care. No history is available. His vital signs are essentially stable, finger stick blood sugar is 111 mg/dL, there are no signs of trauma, and no physical findings consistent with common drug or alcohol use. A toxicology screen is pending. The AGACNP orders acute psychiatric stabilization with a combination of haloperidol and lorazepam and considers which of the following mediations to decrease the risk of adverse effects?

 

A.

Risperidone

 

B.

Olanzapine

 

C.

Benztropine

 

D.

Zolpidem

QUESTION 75

1. Amy is a 21-year-old female who presents with acute nephrolithiasis. CT scan reveals a 2 mm stone in the left ureter. The AGACNP knows that the appropriate course of action is:

 

A.

Pain control and IV fluid

 

B.

Consultation for stent placement

 

C.

Lithotripsy stone destruction

 

D.

Transurethral stone destruction

QUESTION 76

1. B.T. is a 49-year-old male being admitted for lung volume reduction surgery. His preoperative pulmonary function tests are as follows:

FVC 66% predicted

FEV1 60% predicted

PEFR 69% predicted

TLC 104% predicted

RV 90% predicted

The AGACNP knows that the pulmonary function studies are consistent with:

 

A.

Mild restrictive disease

 

B.

Moderate restrictive disease

 

C.

Mild obstructive disease

 

D.

Moderate obstructive disease

QUESTION 77

1. A 30-year-old male patient presents for evaluation of a lump on his neck. He denies pain, itch, erythema, edema, or any other symptoms. He is concerned because it won’t go away. He says, “I noticed it a few months ago, then it seemed to disappear, and now it is back.” The AGACNP proceeds with a history and physical exam and concludes which of the following as the leading differential diagnosis?

 

A.

Subclinical infection

 

B.

Non-Hodgkin’s lymphoma

 

C.

Catscratch disease

 

D.

Syphilis

QUESTION 78

1. Ms. Teller presents with a chief complaint of weight loss. She reports an unplanned 10 lb weight loss over the last 5-6 months. She has no significant medical history, but review of systems reveals bilateral shoulder discomfort and some impaired range of motion—she has trouble pulling clothing over her head. Over the last few months she has generalized upper body stiffness, but seems to get better after an hour or so of activity. When considering a diagnosis of polymyalgia rheumatica, laboratory assessment may be expected to reveal:

 

A.

An erythrocyte sedimentation rate (ESR) of 75   mm/hr

 

B.

A microcytic, hypochromic anemia

 

C.

Elevated liver function enzymes

 

D.

Positive antinuclear antibodies

QUESTION 79

1. When a patient has lower abdominal discomfort, cervical wall motion tenderness, and adnexal tenderness, the AGACNP knows that this will likely be treated with:

 

A.

Ceftriaxone and azithromycin

 

B.

Metronidazole and ciprofloxacin

 

C.

Trimethoprim/sulfamethoxazole

 

D.

IV fluid and pain control

QUESTION 80

1. J.L. is an 81-year-old female who is admitted from home after her daughter found her confused and unkempt. She is not a good historian, and her daughter cannot provide any information—when she saw her mother a week ago, she was fine. J.L.’s vital signs are as follows: Temperature 101.4° F, pulse 99 b.p.m., respirations 22 b.p.m., and blood pressure 90/58 mm Hg. Her urinalysis is shows +++ leukocytes, + RBC, and + nitrites. Her metabolic panel reveals a BUN of 39 mg/dL and creatinine of 1.5 mg/dL. The AGACNP knows that J.L has findings consistent with:

 

A.

Pre-renal failure

 

B.

Intra-renal failure

 

C.

Post-renal failure

 

D.

Chronic renal failure

QUESTION 81

1. Patients with giant cell arteritis are at increased risk of:

 

A.

Cerebrovascular accident

 

B.

Rheumatoid arthritis

 

C.

Polymyalgia rheumatica

 

D.

Osteoarthritis

QUESTION 82

1. The AGACNP is called to the bedside for a patient who is in cardiopulmonary arrest. The monitor demonstrates ventricular fibrillation which will not convert despite several attempts to defibrillate at maximal voltage. While being briefed by the staff nurse on the patient medical history, he learns that the patient has a history of Cushing’s syndrome. The AGACNP recognizes that the patient is probably failing to convert due to:

 

A.

Advanced atherosclerotic disease

 

B.

Hypokalemia

 

C.

Hypocalcemia

 

D.

Catecholamine excess

QUESTION 83

1. When performing an evaluation of a patient following seizure activity, the AGACNP knows that the most important component of that evaluation is:

 

A.

A CT scan of the head

 

B.

Eyewitness description

 

C.

An EEG

 

D.

Administering a benzodiazepine

QUESTION 84

1. Which of the following etiologic organisms is most likely to appear as lobar consolidation on chest radiography?

 

A.

Legionella pneumophilia

 

B.

Streptococcus pneumoniae

 

C.

Pneumocystis carinii

 

D.

Mycoplasma pneumoniae

QUESTION 85

1. A patient with chronic kidney disease presents with an eGFR of 30 mL/min/1.73m2. The AGACNP knows that the most compelling implication of this value is:

 

A.

Control of risk factors for renal deterioration

 

B.

Careful attention to renal dosing of medications

 

C.

Referring the patient for shunt placement

 

D.

Preventing occurrence of renal ischemia

QUESTION 86

1. Differential diagnosis of hematuria include all of the following except:

 

A.

Bladder cancer

 

B.

Nephrolithiasis in the renal parenchyma

 

C.

Urinary tract infection

 

D.

Prerenal azotemia

QUESTION 87

1. A 29-year-old male patient presents with acute scrotal pain and dysuria. He has a temperature of 101.8° F and a pulse of 115 b.p.m. but otherwise vital signs are within normal limits. He gets some relief of the scrotal discomfort when his scrotum is elevated on a rolled towel. This is known as:

 

A.

Varicocele

 

B.

Prehn’s sign

 

C.

Cremasteric sign

 

D.

Testicular torsion

QUESTION 88

1. The AGACNP knows that patients with psoriasis are at greater risk for:

 

A.

Arthritis

 

 

B.

Eczema

 

C.

Cellulitis

 

D.

Melanoma

QUESTION 89

1. Mr. McCarran is a 68-year-old male with a long history of poorly controlled T2DM. He has had progressive burning pain in both feet for the last year or so, but in the last few months it has become increasingly worse. He has tried taking ibuprofen and naproxyn over-the-counter with no improvement. Now, he is presenting for more effective pain management. The AGACNP knows that the medication of choice will be from which drug class?

 

A.

NSAIDs

 

B.

Opiates

 

C.

Antiepileptics

 

D.

Anesthetics

QUESTION 90

1. Mr. Starwood is a 61-year-old male who was admitted last night for the management of acute pancreatitis. He was admitted n.p.o and started on intravenous fluid and opiate pain management. This morning he reports feeling significantly better. His C-reactive protein this a.m. is 5 mg/dL, amylase and lipase are both just over 2 x upper limits of normal, and his Ransom score is 2. The AGACNP knows that the next step in his care is to:

 

A.

Begin clear liquids as tolerated

 

B.

Order an abdominal CT

 

C.

Order an ERCP

 

D.

Continue the current management for 24 hours

QUESTION 91

1. When beginning pharmacotherapy for depression, the AGACNP discusses with the patient that a primary safety consideration includes the:

 

A.

Increased risk of suicide when patients begin   antidepressant therapy

 

B.

Potential for sexual adverse effects

 

C.

Better likelihood of success when medications and   therapy are used together

 

D.

High incidence of serotonin syndrome

QUESTION 92

1. When ruling out meningitis in a patient, the AGACP appreciates that the spinal fluid is cloudy and the glucose content is 20 cells/microliter. This is most consistent with:

 

A.

Aseptic meningitis

 

B.

Septic meningitis

 

C.

Chemical meningitis

 

D.

Chronic meningitis

QUESTION 93

1. A 44-year-old male patient presents in a hypertensive crisis. The blood pressure is 240/136 mm Hg, pulse is 128 b.p.m. and the patient is complaining of a severe, pounding headache. His skin is diaphoretic and he is visibly tremulous. The first diagnostic study to evaluate the suspected diagnosis should be a:

 

A.

24 hour urine for catecholamine metabolites

 

B.

Serum epinephrine and metanephrines

 

C.

T scan of the abdomen

 

D.

MRI of the abdomen

QUESTION 94

1. An unidentified patient is brought to the emergency department by ambulance after being hit by a motor vehicle. She has multiple injuries and an estimated blood loss of 2 liters. The hematocrit is 19%. The AGACNP expects that the mean cell volume (MCV) would most likely be:

 

A.

70 fL

 

B.

80 fL

 

C.

90 fL

 

D.

110 fL

QUESTION 95

1. Mrs. Oliver is a 71-year-old petite Caucasian female. During a routine dexa screening she was found to have a T-score of -3.0. The AGACNP knows that the first intervention should include:

 

A.

Calcium

 

B.

Vitamin D

 

C.

Bisphosphonates

 

D.

Estrogen

QUESTION 96

1. When evaluating a family with suspected carbon monoxide exposure, the AGACNP knows that assessment should include all of the following except:

 

A.

Vital signs

 

B.

Pulse oximetry

 

C.

Cardiac rhythm strip

 

D.

Carboxyhemoglobin level

QUESTION 97

1. Mr. Riley is a 61-year-old male who just had bilateral knee replacements. There was more fluid loss than intended during the procedure. The AGACNP knows that metabolic alkalosis is the most common postoperative acid-base imbalance and is best treated with:

 

A.

Normal saline infusion

 

B.

An insulin drip

 

C.

Low volume hydrochloric acid

 

D.

Albumin

QUESTION 98

1. Patients in advanced stages of chronic kidney disease are at greatest risk for which of the following conditions?

 

A.

Polycythemia

 

B.

Hypokalemia

 

C.

Metabolic alkalosis

 

D.

Anemia

QUESTION 99

1. The diagnostic study of choice in mesenteric ischemia is:

 

A.

Ultrasound

 

B.

CT angiography

 

C.

MR angiography

 

D.

Diagnostic peritoneal lavage

QUESTION 100

1. Mr. Maxwell is a 58-year-old male who presents with left foot pain. Physical examination reveals a foot that is normal in appearance with DP and PT pulses that are barely audible by Doppler. The AGACNP has the patient cross the leg with the left foot resting on the right knee; after 30 seconds that left foot is briskly lowered to the floor. Instantly the left foot turns bright red. This is known as:

 

A.

Venous insufficiency

 

B.

Brawny hyperpigmentation

 

C.

Homan’s sign

 

D.

Dependent rubor

QUESTION 101

1. 152: When completing this exam, did you comply with Walden University’s Code of Conduct including the expectations for academic integrity?

 

Yes

 

No

 

NURS 6550 Final Exam / NURS6550 Final Exam (Latest): Walden University

Walden University NURS 6550 Final Exam / Walden University NURS6550 Final Exam

Question 

A 21-year-old woman requests hormonal emergency contraception after a condom break during intercourse approximately 16 hours ago. Today is day 14 of her normally 27–29 day menstrual cycle. You advise her that:

Likelihood of conception is minimal and emergency contraception use is not advised

Hormonal emergency contraception can be effective up to 5 days after intercourse

When taken as advised, hormonal emergency contraception use reduces the risk of pregnancy by up to 65%

The most likely mechanism of action of hormonal emergency contraceptive is as an abortifacient

Question 

A 36-year-old man is hospitalized for alcohol poisoning. During follow-up evaluation the next day, he denies that he has a drinking problem. The best approach to assess for possible alcoholism is conducting which of the following questionnaires?

BDI.

HEADDS.

CAGE.

PHQ-9.

Question 

A quality improvement plan characterized by limiting variability and removing defects in a process best describes:

Six Sigma.

Continuous Quality Improvement.

Total Quality Management.

Healthcare Stewardship.

Question 

A 38-year-old woman comes to the emergency department complaining of a rapid heartbeat, tremors, and chest tightness. She reports earlier in the day she was feeling a migraine starting so she took a pill given to her by her friend who also experiences migraines. Her medical history reveals that she is currently taking a monoamine oxidase inhibitor for depression. Which migraine medication did she most likely take?

Acetaminophen and caffeine (Excedrin ® Migraine)

Naproxen sodium (Aleve ® )

Almotriptan (Axert ® )

Butalbital, acetaminophen and caffeine (Fioricet ® )

Question 

All of the following are elements of malpractice except:

A duty of care to the patient.

A fee is charged for the healthcare services related to the malpractice claim.

Breach of the standard of care.

Injury.

Question 

You are seeing a 68-year-old woman for treatment of an uncomplicated urinary tract infection (UTI). She has well-controlled hypertension, type 2 diabetes mellitus, and dyslipidemia and takes an angiotensin-converting enzyme inhibitor (ACEI), statin, biguanide, and low-dose aspirin (ASA). She worked in a dry cleaning facility until approximately 8 years ago. During her evaluation, she mentions that she sometimes has difficulty understanding conversation, especially in noisy environments. This is likely a:

Drug-related reaction.

Consequence of occupational chemical exposure.

Early sign of dementia.

Normal age-related change in hearing

Question 

While evaluating a 33-year-old female with a 2-day history of dysuria, which of the following findings in urinalysis is most suggestive of urinary tract infection (UTI) caused by a Gram-negative organism?

Nitrites

30 mg/dL protein

Epithelial cells

pH>8

Question 

All of the following are examples of primary prevention strategies except:

Counseling an elderly patient prior to discharge about fall risk at home and how to prevent falls through adequate illumination.

Performing a hemoglobin A1C for all patients admitted to the hospital.

Conducting a study to identify the leading cause of mortality in teenagers and how to reduce risk.

Immunizing all adults ≥60 years with zoster vaccine.

Question 

A 23-year-old woman is being evaluated for an upper respiratory tract infection. As you prepare for auscultation, the patient states “I have a benign murmur that has been with me my whole life”. Anticipating a physiologic murmur, you would expect which of the following characteristics?

Usually obliterates S2.

Becomes softer when going from a supine to standing position.

Occurs late in systole.

Has localized area of auscultation

Question 

When evaluating the value of a test, the proportion of negative results that are truly negative best describes:

Positive predictive value.

Negative predictive value.

Sensitivity.

Specificity.

Question 

A 16-year-old male presents for evaluation after a syncopal episode at school. He reports that he has recently been experiencing bouts of lightheadedness. His blood pressure is 126/76 mm Hg and his BMI=33 kg/m 2 . You notice velvet-like plaques at the nape of the neck. Laboratory assessment should include which of the following tests?

Hemoglobin A1c

Serum electrolytes

ALT/AST

Hemogram

Question 

The NP is called to treat a 43-year-old woman for multiple contusions following a domestic violence attack. The NP realizes that one of the best predictors of a subsequent homicide of victims of domestic violence is:

History of alcohol/drug abuse by perpetrator.

Access to kitchen knives by the perpetrator.

History of perpetrator attempting to strangle the victim.

Issuance of a restraining order by the victim.

Question 

A 48-year-old woman is being evaluated with chief complaints of fatigue, weakness, lethargy, and decreased concentration. She also mentions a notable increase in facial hair over the past 6 months as well as unexplained weight gain (about 25 lbs [11.3 kg]) over the past 2 months. Her past medical history is notable for moderate persistent asthma with multiple exacerbations over the past 8 months requiring treatment with prednisone. The most likely diagnosis is:

Type 2 diabetes mellitus.

Hashimoto thyroiditis.

Cushing’s syndrome.

Addison’s disease.

Question 

You see a 73-year-old woman with a 40 pack-year smoking history, chronic obstructive pulmonary disease (COPD) and peripheral arterial disease (PAD) who presents with an ulcer on the sole of her left foot. The ulcer has an irregular edge and pale base and a punched out appearance, with the surrounding skin white and shiny. The patient states that the pain is worse at night in bed and when the legs are elevated. The most likely diagnosis is:

Pressure ulcer

Pyoderma gangrenosum

Venous ulcer

Arterial ulcer

Question 

In evaluating a 62-year-old male with ischemic heart disease and mitral incompetency, you expect to find the murmur that is:

Localized systolic.

Diastolic with radiation to the neck.

Diastolic with little radiation.

Systolic with radiation to the axillae.

Question 

21-year-old camp counselor presents for evaluation with a chief complaint of generalized itchiness. He has just returned after a week of summer camp. Physical examination reveals excoriated papules along his axillary folds and at the belt line. The NP suspects scabies and prescribes permethrin (Elimite ® ) lotion. In counseling the patient on the use of permethrin, the NP mentions that:

Its use is associated with neurotoxicity risk.

Itch often persists for a few weeks after successful treatment.

A marked reduction in lesions is noted within 48 hours.

The medication should be rinsed off within 2 hours of application.

Question 

You see a 76-year-old woman living at home who is accompanied by her home care provider. She has COPD and type 2 diabetes mellitus. An example of a secondary prevention strategy is:

Administering the seasonal influenza vaccine.

Screening for physical or financial abuse/Checking her blood glucose level.

Checking her blood pressure.

Adjusting her insulin dosing regimen.

Question 

A 43-year-old woman is being evaluated in the emergency department with a complaint of a severe headache. She describes a unilateral, pulsing headache that was preceded by a gradual onset of paresthesia affecting the ipsilateral face and arm. The patient stated she noticed a “funny smell” prior to the symptoms starting. This description is most typical of:

Migraine with aura.

Cluster headache.

Transient ischemic attack.

Tension-type headache.

Question 

A 20-year-old college student living in a campus dormitory has been diagnosed with meningococcal meningitis. The NP decides to speak with the university health officials and inform them that:

There is little to no risk of spread of the disease to other individuals.

Only intimate partners are at risk for infection.

Individuals with household-type or more intimate contact are at risk for infection.

All individuals on campus can be considered to be at risk for infection.

Question 

You see a 54-year-old man living in subsidized housing with a history of hypertension. He states that he stopped taking his blood pressure medication about 4 months ago because of costs. He is concerned because his blood pressure is high whenever he checks it, though he does not report any symptoms. His BP at this visit is 196/104 mm Hg. Upon examination of this patient, you would expect to detect an S 4 sound heard during:

Early systole.

Late systole.

Early diastole.

Late diastole.

Question 

32-year-old man requires evaluation for hepatitis infection prior to taking a job as a restaurant cook. He presents with the following laboratory results:

Hepatitis A Panel Hepatitis B Panel Hepatitis C Panel Anti-HAV Negative HBsAg Negative Anti-HCV Positive IgM Negative Anti-HBc Negative HCV RNA Negative IgM Negative Anti-HBs Positive You recognize the patient is susceptible to:

Hepatitis A and B

Hepatitis B and C

Hepatitis B only

Hepatitis A only

Question 

When evaluating illness symptoms in older patients, the disease will often present differently from younger adults due to:

The likelihood of polypharmacy.

Decreased compensatory mechanisms in the elderly.

An increased physiologic response to illness in the elderly.

Presence of comorbid conditions.

Question 

When prioritizing risk factors for bacterial endocarditis, the nurse practitioner knows who among the following patients has the highest risk.

A 55-year-old man with 3-vessel coronary artery bypass grafts with stents.

A 23-year-old woman with mitral valve prolapse without tissue redundancy.

A 65-year-old man with nonobstructive cardiomyopathy.

A 75-year-old woman with a nonorganic prosthetic aortic valve.

Question 

The legal authority for NPs to perform healthcare services as defined by state law is called:

Duty of care.

Non-malfeasance.

Autonomy.

Scope of practice.

Question 

A patient who was seen for an upper respiratory tract infection has an abnormal blood test result and requires a follow-up visit. The patient repeatedly fails to show up for the follow-up visit. Which of the following is the best approach to inform the patient of the need and urgency for a repeat test?

A voicemail or text message.

A visit to her home.

An e-mail message marked as “urgent”.

A certified letter.

Question 

A 26-year-old male presents with a chief complaint of bilateral, intermittent itchy eyes accompanied by rope-like discharge. This is most consistent with conjunctivitis caused by:

A virus.

Bacteria.

Over-exposure to smoke.

Allergen.

Question 

When developing a management plan for patients with genitourinary infections, which of the following would you most likely recommend for follow-up imaging following resolution of their infection?

A 27-year-old nonpregnant woman with acute, uncomplicated urinary tract infection (UTI).

A 38-year-old nonpregnant woman with 2 acute, uncomplicated UTIs in the past 9 months.

A 57-year-old man with acute bacterial prostatitis.

A 43-year-old man with type 2 diabetes mellitus and recurrent pyelonephritis.

Question 

s. Wang is a 56-year-old female who was seen 2 weeks ago for evaluation of ongoing abdominal discomfort and nausea. Laboratory assessment revealed H. pylori and she was treated for peptic ulcer disease with an appropriate antibiotic/proton-pump inhibitor combination. She returns today and admits to no real change in her symptoms. This suggests that:

Treatment was inadequate and a salvage regimen should be used.

It is too soon to expect a response and the patient should be reevaluated in four weeks.

The patient could have complicated peptic ulcer disease and should be referred for endoscopy.

Peptic ulcer disease is likely not the correct diagnosis.

Question 

The NP is called to evaluate Jane, a 43-year-old woman, who presents with a diffuse maculopapular rash that began on the trunk but now covers the entire body, including the palms and soles. The rash is not itchy but wart-like sores are present in the mouth and genital area as well as skin folds. Initial evaluation reveals generalized lymphadenopathy and a low-grade fever, and she complains of lethargy and headache. The patient reports that she had a painless genital ulcer that healed on its own about 3 weeks earlier. Which of the following is the most likely diagnosis?

Primary syphilis.

Secondary syphilis.

Genital herpes.

Gonorrhea.

Question 

A 27-year-old man presents with a chief concern of an abnormal mass within his left scrotum. He describes it as feeling like a “bag of worms” and is present when he is standing but disappears when he lies down. His past medical history is unremarkable and his BMI is 29 kg/m 2 . He has been in a monogamous relationship for the past 4months. The most likely diagnosis is:

Testicular torsion.

Syphilis.

Varicocele.

Testicular cancer.

Question 

An 18-year-old man is being evaluated for a severe exacerbation of asthma. He is currently taking an inhaled corticosteroid, a long-acting beta-agonist, and a short-acting beta-agonist on an as needed basis. The most important component of the initial assessment includes a(n):

Continuous pulse oximetry reading

Peak expiratory flow reading

Chest X-ray

Arterial Blood Gas

Question 

A 67-year-old man is being evaluated for shortness of breath. His medical history reveals that he experienced a myocardial infarction about 2 years ago and a history of hypertension, which is well controlled by diet.. Otherwise, his medical history is unremarkable. Which of the following would you expect to find on electrocardiogram (ECG)?

T wave inversion.

Pathologic Q wave.

ST segment elevation.

Tall R wave.

Question 

A 46-year-old man is rushed to the emergency department while experiencing an acute adrenal crisis that presents with abdominal pain, severe vomiting, and low blood pressure. He appears cyanotic and confused. The most appropriate treatment for this patient is an injection of:

Insulin.

Epinephrine .

Hydrocortisone.

Antihistamine.

Question 

A 74-year-old male with asymptomatic atrial fibrillation underwent electrical cardioversion that successfully restored the heart’s rhythm. Which of the following medications can be considered for the purpose of maintaining the heart’s rhythm?

Sotalol (Betapace ® )

Digoxin (Lanoxin ® )

Dabigatran (Pradaxa ® )

Enalapril (Vasotec ® )

Question 

In considering the use of an inhaled anticholinergic, such as tiotropium bromide (Spiriva ® ), for the treatment of chronic obstructive pulmonary disease (COPD), the NP recognizes which of the following is the desired therapeutic action?

Mucolytic agent.

Bronchodilator.

Increases mucociliary clearance.

Antihistamine effect.

Question 

A 14-year-old male is brought in for evaluation after he reportedly collapsed during a tennis match, though he quickly regained consciousness. His medical history is otherwise unremarkable. You suspect hypertrophic cardiomyopathy. Which of the following heart murmur characteristics would support the diagnosis?

Occurs late in systole.

Widely split S₂.

Becomes louder when going from a supine to standing position.

Murmur follows mid-systolic click.

Question 

You see a 64-year-old man with an area of erythema concentrated on the left side of his neck with clusters of vesicles forming a line. He reported pain in the area a couple of days before the eruption of the lesions. He states that he recently initiated biologic therapy for rheumatoid arthritis. Which of the following is the most likely diagnosis?

Impetigo.

Herpes zoster.

Drug-related adverse reaction.

Viral exanthem.

Question 

Gary is a 63-year-old African American male who has been treated for hypertension with ACE inhibitor monotherapy for the past 6 weeks. Despite Gary’s insistence that he is taking his medication as directed, his blood pressure continues to be elevated. Gary mentions that he prefers not having to take multiple medications each day due to cost and convenience. Which of the following is the most appropriate course of action?

Continue with the current regimen as the full effects are not observed until about 8 weeks of treatment.

Add an angiotensin receptor blocker (ARB) to the regimen.

Switch to a beta blocker.

Switch to a calcium channel blocker.

Question 

An NP’s duty of care can be established:

Only in the setting of a healthcare institution (e.g., hospital, clinic, etc.).

When the NP gives professional advice or treatment in any setting.

Only when a fee is charged, either to the patient or third-party payer, for services.

Only when both the NP and patient acknowledge a patient-provider relationship.

Question 

The NP is called to evaluate a 48-year-old man who was brought to the emergency department after experiencing severe low back pain following an attempt to lift heavy furniture. He also reports a coincident loss of bowel and bladder control. This most likely indicates:

Sciatic nerve entrapment

Vertebral fracture

Cauda equina syndrome

Muscular spasm

Question 

A 49-year-old woman presents with a 3-day history of burning during urination, a thin and grayish-white vaginal discharge, and vagina itching. On laboratory examination, you expect to find all of the following except:

Abundant white blood cells

Clue cells

Pseudohyphae

Alkaline vaginal pH

Question 

The NP is called to evaluate a 34-year-old nonpregnant woman who complains of a 4-week history of anxiety, palpitations, diarrhea, unexplained weight loss, and sensitivity to heat. Her medical history is unremarkable and she is not taking any medications. Physical examination reveals warm, moist skin and exaggerated deep tendon reflexes. The NP suspects Graves’ disease. Which of the following laboratory results would best support this diagnosis?

Free T₄ =6 pmol/L (NL=10–27 pmol/L).

Thyroid stimulating hormone (TSH)=0.05 mU/L (NL=0.15–4.0 mU/L).

ESR=37 mm/h (NL <15 mm/h).

Total WBC=4,200/mm³, 10% Neutrophils w/ hypersegmentation (NL=6,000–10,000/mm³ , 50%–70% Neutrophils).

Question 

Phyllis is a 34-year-old woman who presents for a chief complaint of a migraine resulting in nausea and vomiting. She reports that she experiences multiple migraines each month. She is given a prescription for sumatriptan (Imitrex ® ) 100-mg tablets orally to treat acute migraine pain. When counseling Phyllis about the medication, the nurse practitioner advises about all of the following adverse effects except:

Sedation

Chest tightness

Dry mouth

Nausea/vomiting

Question 

In managing a 58-year-old woman who is admitted for deep vein thrombosis, caution should be used with which of the following medications due to a risk of drug-induced thrombocytopenia?

Warfarin (Coumadin®)

Clopidogrel (Plavix®)

Dabigatran (Pradaxa®)

Unfractionated heparin

Question 

A 77-year-old woman with urinary incontinence is brought in by her caregiver for evaluation of a potential urinary tract infection. She is mentally alert and responds clearly to questions. The caregiver waits in the waiting area. While performing a routine comprehensive physical examination, you notice bruises on her right and left upper arms. The most appropriate approach would be to say:

“Did someone grab you here?”

“I’m concerned about your safety regarding these bruises on your arms.”

“I notice you have similar bruising on both arms.”

“Shall I ask your caregiver about these bruises?”

Question 

A 27-year-old woman with a known sulfa allergy presents with an uncomplicated UTI. She has not received any systemic antimicrobials in the past 6 months. She is currently not pregnant and is using norelgestromin/ethinyl estradiol patch (Ortho Evra ® ) for birth control. You recommend treatment with:

Trimethoprim-sulfamethoxazole (Bactrim ® ).

Amoxicillin (Amoxil ® ).

Nitrofurantoin (Macrobid ® ).

Cephalexin (Keflex ® ).

Question 

You are examining a 64-year-old woman with a history of rheumatic heart disease. In assessing the patient for mitral stenosis, you expect to find a heart murmur characterized as:

High-pitched blowing systolic murmur heard best a third left intercostal space.

Late systolic crescendo murmur with honking quality heard best at apex.

Localized low-pitch late diastolic heard best at apex.

Early diastolic murmur heard best at left sternal border with radiation to the neck.

Question 

21-year-old female student presents at the university clinic with a nosebleed. This is her third bleeding episode in the past week. She is otherwise healthy with no history of bleeding disorders. The NP advises that the appropriate first-line intervention for anterior epistaxis is:

Nasal packing.

Application of topical thrombin.

Firm pressure to the area superior to the nasal alar cartilage.

Utilization of a dehumidifier at home.

Question 

Tertiary prevention activities for a 69-year-old woman with congestive heart failure and type 2 diabetes mellitus include:

Adjusting therapy to minimize dyspnea.

Administering the pneumococcal vaccine.

Skin survey for pre-cancerous lesions.

Assessing her creatinine clearance.

Question 

You see a 77-year-old male for treatment of a laceration on his forearm. Upon examination, you notice poor skin turgor when assessed on the arms and back of the hands. This is likely a sign of:

Dehydration.

Normal age-related change.

Early heart failure.

Renal dysfunction.

Question 

All of the following practices are recommended when prescribing medications with confusing or similar-sounding names except:

Writing the purpose of the medication on the prescription order.

Considering selecting medications without nomenclature problems.

Including either the generic or brand name on the prescription order, but not both.

Providing patients with written information about their drugs.

Question 

Which of the following describes the ethical principle of veracity?

Healthcare resources are allocated so that the best is done for the greatest number of people.

The responsibility of the healthcare provider is to treat all people in the same fair manner.

The healthcare provider must be truthful and avoid deception.

The healthcare provider has an obligation to be faithful to commitments made to self and others.

Question 

Which of the following examples describes a potential malpractice scenario?

A patient with known penicillin allergy is prescribed amoxicillin but no allergic reaction occurs.

A post-myocardial infarction patient is prescribed an inappropriate dose of clopidogrel and experiences a severe bleeding episode.

A patient with a urinary tract infection does not see any improvement in signs and symptoms 3 days after given a dose-appropriate prescription for trimethoprim-sulfamethoxazole.

A patient realizes prior to taking any of the medication that the wrong drug was dispensed at the pharmacy.

Question 

An 84-year-old female patient is admitted from a local long-term care facility (illnes). The patient is normally awake, alert, and oriented. She resides in the LTCF because she has not fully recovered from a broken hip resulting from a fall 4 months ago; otherwise she is in relatively good health. She can walk short distances with a walker, though she primarily stays in a wheelchair. Today, however, the nursing staff found her to be acutely confused and unable to ambulate without falling. She was transferred to the emergency department for evaluation. Initial laboratory testing must include:

2 sets of blood cultures.

Serum thyroid stimulating hormone (TSH) level.

Urinalysis.

Lumbar puncture.

Question 

A 67-year-old female has a long history of chronic venous insufficiency (CVI) and was seen by the nurse practitioner for brown discoloration on her calves. The nurse practitioner diagnoses hyperpigmentation due to CVI and presented the appropriate treatment plan. When evaluating the patient’s understanding, the nurse practitioner expects that she will say:

“I should stay out of the sun or use sunscreen when my legs must be exposed.”

“I need to elevate my legs as often as I can.”

“I will use hydrocortisone cream twice a day for the next two weeks and then come back for my appointment.”

“There is really nothing to help this discoloration.”

Question 

The NP is called to evaluate a 56-year-old man complaining of severe pain in the upper right abdomen that radiates to the right shoulder, nausea and vomiting, which started soon after dinner. The NP suspects acute cholecystitis. All of the following findings would be consistent with the diagnosis except:

Elevated aminotransaminase (AST)

Elevated alkaline phosphatase (ALP)

Microcytic anemia

Leukocytosis

Question 

A 43-year-old woman presents in the emergency department with a chief complaint of swelling of the ankles, hands, and face. She also reports urine that is darkly colored and foamy. Her blood pressure is 184/118 mm Hg. All of the following urinalysis findings will support a diagnosis of glomerulonephritis except:

Elevated levels of ketones.

Presence of white blood cells.

Elevated levels of protein.

Presence of red blood cell casts.

Question 

While conducting an examination of a 15-year-old female accompanied by her mother regarding a potential sexually transmitted disease, the NP knows that the best approach to the visit would be to:

Ensure the mother is present at all times.

Ask the adolescent if she wishes the mother to be included in the interview and examination.

Interview the adolescent with the mother and then asking the mother to leave for the physical examination.

Interview and examine the adolescent in the absence of the mother.

Question 

A 23-year-old college student presents with a 2-day history of severe sore throat and difficulty eating or drinking due to trouble swallowing. A physical examination reveals exudative pharyngitis and minimally tender anterior and posterior cervical lymphadenopathy. The NP suspects infectious mononucleosis and would expect which of the following laboratory findings?

Neutrophilia with reactive forms.

Thrombocytosis.

Lymphocytosis with atypical lymphocytes.

Diminished ALT/AST levels.

Question 

A 45-year-old male is being prepared for release after receiving analgesic treatment for low back pain due to an acute lumbosacral strain. He is in otherwise good health and typically exercises on a daily basis. He asks when he will be able to start exercising again. The most appropriate response is:

You should not exercise until you are completely pain-free.

Conditioning exercises should be started immediately.

Leg numbness is to be expected with muscle-strengthening exercises.

Back-strengthening exercises may cause mild muscle soreness.

Question 

Mrs. Gonzalez is a 53-year-old female who is being evaluated for routine follow-up care. Her medical history includes dyslipidemia, hypertension, and rheumatoid arthritis. Laboratory analysis reveals elevated serum alanine transaminase (ALT) and aspartate transaminase (AST). The long-term use of which of the following medications is the most likely cause of this finding?

Calcium channel blocker

Statin

ACE inhibitor

Methotrexate

Question 

“Incident-to” services are defined as those which are “an integral, although incidental, part of the physician’s personal professional services in the course of diagnosis or treatment of an injury or illness”. As long as certain criteria are met, “incident to” billing is an option in all of the following settings except:

Office visits.

Hospital.

Nursing home.

Home visits.

Question 

A 78-year-old male is being treated for a hypoglycemic episode. He has a long history of type 2 diabetes mellitus as well as congestive heart failure and COPD. He is currently taking basal insulin with a sulfonylurea. What is the acceptable A1C goal for this patient?

≤6.5%

≤7.0%

≤8.0%

≤9.0%

Question 

A 47-year-old woman reports that her long-time boyfriend was recently diagnosed with liver cancer possibly caused by a chronic hepatitis B infection. Though she does not have any symptoms, she wants to be checked for hepatitis B. She can’t recall if she has ever been vaccinated for hepatitis B. The laboratory results are as follows:

Hepatitis B surface antigen (HBsAg): Positive Hepatitis B surface antibody (Anti-HBs): Negative Total hepatitis B core antibody (Anti-HBc): Positive IgM antibody to hepatitis B core antigen: Negative These findings are most consistent with:

Evidence of effective hepatitis B immunization.

Evidence of hepatitis B infection in the past.

Immunity against future hepatitis B infection.

Chronic hepatitis B.

Question 

You see a 74-year-old male who is accompanied by his granddaughter who lives with him. He has hypertension, a prior myocardial infarction, long-standing type 2 diabetes, and recently underwent a lower limb amputation secondary to diabetes. He is currently taking an ACE inhibitor, statin, low-dose aspirin, biguanide, and insulin. The granddaughter reports that since the amputation, her grandfather sometimes becomes withdrawn, irritable and moody for no apparent reason, does not want to participate in the typical activities he enjoys, and often does not appear to eat much during the day. She also states that he often complains of being tired but normally wakes up in the early morning hours. The most appropriate action is to evaluate the patient for:

Dementia.

Depression.

Delirium.

Drug interaction.

Question 

A 15-year-old male is brought in to the emergency department by his parents following a 12-hour history of nausea, vomiting, and abdominal pain. Physical examination reveals obturator and psoas signs and a temperature of 102.6ºF (39.2ºC). Anticipated white blood cell (WBC) with differential results are as follows:

Total WBC=18,100/mm 3 , Neutrophils=50%, Bands=1%, Lymphocytes=40%

Total WBC=14,000/mm 3 , Neutrophils=55%, Bands=3%, Lymphocytes=38%

Total WBC=4,500/mm 3 , Neutrophils=35%, Bands=2%, Lymphocytes=45%–55% with reactive forms

Total WBC=16,500/mm 3 , Neutrophils=66%, Bands=8%, Lymphocytes=22%

Question 

A 42-year-old male presents with a sudden onset of inability to tightly close the eye lid, frown, or smile on the right side. His examination is otherwise unremarkable and past medical history is unremarkable. This likely represents paralysis of cranial nerve (CN):

III

IV

VII

VIII

Question 

Which of the following activities best demonstrates fulfillment of the nursing leadership role?

Screening a 36-year-old woman for hypertension.

Collaborating with a local health clinic on strategies to improve adult vaccination rates.

Volunteering to teach a group of high school students about the dangers of prescription drug abuse.

Collaborating with an infectious disease specialist on the care of a patient with community-acquired pneumonia.

Question 

In considering treatment for Jane in the previous question, which of the following would be most appropriate if she has a history of penicillin allergy?

Amoxicillin (Amoxil ® ).

Doxycycline (Doryx ® ).

Famciclovir (Famvir ® ).

Ciprofloxacin (Cipro ® ).

Question 

A 17-year-old high school wrestler presents with an 8 cm-diameter area of warm, red, edematous area on his left arm with sharply demarcated borders. The patient is otherwise healthy and without fever. Given a concern for community-associated methicillin-resistant Staphylococcus aureus (CA-MRSA) infection, the most appropriate treatment option for this patient is:

High-dose amoxicillin (Amoxil ® ).

Amoxicillin-clavulanate (Augmentin ® ).

Trimethoprim-sulfamethoxazole (Bactrim ® ).

Daptomycin (Cubicin ® ).

Question 

You see a 58-year-old man who complains of a persistent dry, hacking cough. He reports that he recently started taking a medication to treat high blood pressure. He is most likely taking a(n):

Angiotensin-converting enzyme (ACE) inhibitor

Alpha-adrenergic antagonist

Angiotensin receptor blocker

Beta-adrenergic antagonist

Question 

A 77-year-old woman is accompanied by her husband for evaluation. She is currently being treated with metoprolol (Lopressor ® ) for hypertension and digoxin (Lanoxin ® ) for atrial fibrillation. He reports that his wife is becoming increasingly forgetful over the past year, failing to note important family events such as birthdays, and sometimes becomes confused with driving directions to familiar locations. She has no previous psychiatric history. The most likely diagnosis is:

Parkinson’s disease.

Delirium.

Dementia.

Early stage of congestive heart failure.

Question 

The NP is called to evaluate a 51-year-old man with acute bacterial rhinosinusitis. He currently smokes 1 PPD and has a 30 pack-year cigarette smoking history. His medical history shows that he is allergic to penicillin. Two weeks ago, he was treated with a macrolide for “bronchitis”. You now prescribe:

Clarithromycin (Biaxin ® ).

Levofloxacin (Levaquin ® ).

Cephalexin (Keflex ® ).

Amoxicillin-clavulanate (Augmentin ® ).

Question 

Which of the following is most consistent with the hepatic enzyme profile of a person with non-alcoholic fatty liver disease?

AST=1208 U/L, ALT=560 U/L.

AST=45 U/L, ALT=88 U/L.

AST=678 U/L, ALT=990 U/L.

AST=98 U/L, ALT=149 U/L.

Question 

A 32-year-old woman in her second trimester of pregnancy presents with fever, flank pain, and hematuria. The NP suspects pyelonephritis. Which of the following urinalysis results would be most indicative of pyelonephritis?

>100 red blood cells (RBCs) per high power field (HPF)

Protein ≥300 mg/dL

White blood cell (WBC) casts

Ketones

Question 

Laboratory findings of a 28-year-old male suspected of heatstroke can include all of the following except:

Hyperkalemia.

Elevated total creatine kinase.

Hypernatremia.

Leukocytosis.

Question 

Mrs. Conner is a 76-year-old woman living in a long-term care facility and has been bedridden with a respiratory infection for the past 4 days. She is brought in for evaluation and you note signs of dehydration as well as a section of epidermal skin loss about 3 cm in diameter on her right hip. The dermal layer appears intact. This would be considered a pressure ulcer of Stage:

1

2

3

4

Question 

Which of the following represents the highest level of scientific evidence when evaluating clinical research?

A randomized controlled trial.

Systematic review/ Meta-analysis of randomized controlled troals.

Observational study.

Cohort study.

Question 

A 24-year-old with a history of major depressive disorder is rushed to the emergency department by his friends after taking an overdose of antidepressant medication. The ingestion of which of the following medications poses the greatest risk of death for this patient?

A 4-day supply of diazepam (Valium ® )

A 3-week supply of Bupropion (Wellbutrin ® )

A 2-week supply of nortriptyline (Pamelor ® )

A 3-week supply of duloxetine (Cymbalta ® )

Question 

The NP is called to evaluate a 72-year-old woman with community-acquired pneumonia. Her vital signs include temperature 101.6ºF (38.7ºC), BP 106/68 mm Hg, and heart rate 78 bpm. Physical examination reveals increased tactile fremitus and dullness to percussion at the left lung base. These findings are consistent with:

Cavitation

Pneumothorax

Lung abscess

Consolidation

Question 

A 64-year-old woman with chronic kidney disease presents with a chief complaint of lethargy. His hemogram is as follows: -Hemoglobin (Hg)=9.9 g/dL (12–14 g/dL -Hematocrit (Hct)=30% (36%–42%) -Mean cell volume (MCV)=81 fL (80–96 fL -Reticulocytes=0.7% (1%–2%) These findings are most consistent with:

Iron deficiency anemia.

Anemia of chronic disease.

Folate deficiency anemia.

Thalassemia trait.

Question 

The NP is called to evaluate a 57-year-old man who reports episodes of acute angina with physical exertion. He is currently taking an ACE inhibitor and low-dose aspirin. The NP considers which of the following approaches at the start of anginal symptoms?

An oral dose of a P2Y 12 inhibitor.

An extra dose of the ACE inhibitor.

A dose of nitroglycerin via sublingual spray.

Supplemental oxygen therapy for 10 minutes.

Question 

You see a 71-year-old man with atrial fibrillation and chronic obstructive pulmonary disease (COPD) who is diagnosed with community-acquired pneumonia. His medical history does not show any drug allergies and he completed a course of a respiratory fluoroquinolone for the treatment of acute bacterial sinusitis within the past month. Which of the following would you consider the most appropriate choice of antimicrobial?

Cefpodoxime (Vantin ® )

Amoxicillin (Amoxil ® ) with doxycycline (Doryx ® )

Azithromycin (Zithromax ® )

Moxifloxacin (Avelox ® )

Question 

A 27-year-old woman is being evaluated for a 7-day history of acute bacterial rhinosinusitis. She is otherwise healthy and has not received any antimicrobial therapy in the past 6 months. She also has no history of drug allergy. In considering a first-line antimicrobial treatment, all of the following would be appropriate except :

Amoxicillin (Amoxil ® ).

Cefuroxime (Ceftin ® ).

Azithromycin (Zithromax ® ).

Levofloxacin (Levaquin ® ).

Question 

When managing elderly patients with diminished cognitive function, which of the following statements is true regarding patient competence for making informed healthcare decisions?

First-degree relatives can declare a person incompetent.

Healthcare providers do not have the ability to determine whether a patient can provide informed consent.

A pattern of impaired judgment can be used to declare a patient incompetent.

Only a court can appoint a guardian to make decisions for a patient declared incompetent.

Question 

n 18-year-old female is being evaluated for wheezing that occurs on an intermittent basis. Symptoms occur about 3–4-times weekly, but not daily. Nighttime awakenings due to wheezing or coughing occur about once per week. Her FEV 1 is 85% predicted and Asthma Control Test (ACT) score is 22. The patient is diagnosed with mild persistent asthma and initiated on Step 2 treatment, which will include:

Short-acting beta 2 -agonist (SABA) on an as needed basis only.

Low-dose inhaled corticosteroid (ICS) only.

Medium-dose ICS plus LABA.

LABA only.

Question 

A 57-year-old female presents for a flu shot as she has heard reports that it will be “a bad flu season”. She has had no primary care for more than 10 years as she lacked health insurance. She admits to a 30 pack-year cigarette smoking history, currently smoking 1 pack-per-day (PPD). Her history and examination are unremarkable. Which screening test will you recommend?

Sputum cytology

Mammography

Hemoglobin electrophoresis

Fasting serum triglycerides

Question 

A 46-year-old woman presents with a 12-hour history of fever, severe pain in the upper right abdomen, and positive Murphy’s sign. Her medical history is unremarkable and she is in otherwise good health. The most likely diagnosis is:

Appendicitis.

Chronic cholelithiasis.

Acute cholecystitis.

Acute viral hepatitis.

Question 

You see a 68-year-old woman who is being treated for moderate depression. She complains that her medication is causing symptoms of dry mouth and constipation. She is most likely taking which of the following medications?

Venlafaxine (Effexor ® )

Citalopram (Celexa ® )

Fluoxetine (Prozac ® )

Nortriptyline (Pamelor ® )

Question 

Bill is a 46-year-old man who is rushed to the emergency department after experiencing sudden onset of edema of the lips and face and a sensation of “throat tightness and difficulty breathing” following a wasp sting. His blood pressure is 78/52 mm Hg, heart rate 120 bpm, and respiratory rate 32/min. The first course of action in caring for Bill is to administer:

Diphenhydramine (Benadryl ® ).

Epinephrine (EpiPen ® ).

Nitroglycerin (Nitrolingual ® ).

Ranitidine (Zantac ® ).

Question 

You are evaluating a 44-year-old woman with community-acquired pneumonia. She has no comorbidities, no reported history of drug allergy, and has not received any antimicrobial therapy in the past 6 months. Which of the following is the most appropriate treatment choice?

Ceftriaxone (Rocephin ® ).

Azithromycin (Zithromax ® ).

Linezolid (Zyvox ® ).

Moxifloxacin (Avelox ® ).

Question 

Bettie is a 33-year-old mother of two children who presents for treatment of a lower urinary tract infection. This is her third episode in the past 6 months. She is prescribed appropriate antibiotic therapy and is taught strategies to decrease urinary tract infection. Which of the following is recommended for the prevention of recurrent UTI in at-risk patients?

Voiding before and after coitus.

Drinking two glasses of cranberry juice each day.

Use of lactobacillus probiotics.

Continuous low-dose antimicrobial prophylaxis.

Question 

A 26-year-old woman presents with uncomplicated UTI. She is otherwise healthy, has not received any systemic antimicrobials in the past year, does not have a sulfa allergy, and is not taking any medications. She is currently not pregnant and is using etonogestrel/ethinyl estradiol vaginal ring (NuvaRing ® ) for contraception. The local E. coli resistance rate to TMP/SMX is about 12%. The preferred therapy for this patient is:

Trimethoprim-sulfamethoxazole (Bactrim ® ).

Amoxicillin (Amoxil ® ).

Nitrofurantoin (Macrobid ® ).

Cephalexin (Keflex ® ).

Question 

A 78-year-old man is brought in by his neighbor who reports that she found him sitting in the front yard. She reports that he mumbled something about waiting for his wife, though she passed away more than 10 years ago. The neighbor says that the patient lives by himself and that this is the first time he has acted in this manner. The patient is calm but has trouble understanding simple directions and is unable to respond promptly to simple questions. Suspecting delirium, the most appropriate first course of action is to:

Administer an antipsychotic.

Refer for neurological consult.

Identify the underlying illness.

Order a head computed tomography (CT) scan with contrast.

Question 

A 19-year-old college freshman has been urged by her friends to be evaluated for lethargy, loss of appetite, and hypersomnia over the past 2 weeks. You administer the Beck Depression Inventory (BDI) assessment and she scores a 22. This would indicate:

No depression.

Mild depression.

Moderate depression.

Severe depression.

Question 

The healthcare principle that promotes actions that lead to achieving the greatest good for the greatest number is known as:

Autonomy.

Utilitarianism.

Beneficence.

Fidelity.

Question 

A 43-year-old man who smokes 1 pack-per-day (PPD) and has an 18 pack-year smoking history presents with a chief complaint of increasing shortness of breath, chronic cough, and sputum production. In considering chronic obstructive pulmonary disease (COPD), which of the following would most support the diagnosis?

FEV₁ >70% predicted.

FEV₁ <85% predicted.

FEV₁/FVC <0.70.

FEV₁ /FVC <1.0.

Question 

A 75-year-old man presents with chief complaints of headache, nausea, and muscle weakness. His creatinine level is 2.1 mg/dL (185.6 μmol/L) and GFR is 30 mL/min/1.73 m 2 . He currently takes a thiazide diuretic for the management of hypertension. Which of the following statements is false regarding the use of diuretics in the elderly?

Thiazide diuretics are less effective when creatinine level is ≥1.8 mg/dL (159.1 mol/L).

Thiazide diuretic use is an independent risk factor for the development of type 2 diabetes mellitus.

Loop diuretics are contraindicated when GFR is ≤40 mL/min/1.73 m².

Thiazide diuretic use leads to low volume sodium depletion resulting in peripheral vascular resistance (PVR) reduction.

Question 

All of the following persons are eligible for Medicare services except:

A 74-year-old ex-smoker with COPD and high income from assets.

A 69-year-old undocumented resident in the US with atrial fibrillation.

A 62-year-old with a permanent physical disability due to a motor vehicle accident.

A 72-year-old permanent legal resident (non-US citizen) with type 1 diabetes mellitus.

Question 

A 35-year-old woman presents with a chief complaint of being unable to close her right eyelid tightly, frown, or smile on the right side. She is otherwise healthy with no significant prior medical history. An appropriate diagnostic test for this patient would be:

Erythrocyte sedimentation rate (ESR).

Alanine transaminase/aspartate transaminase (ALT/AST) levels.

Lyme disease antibody titer.

Thyroid stimulating hormone (TSH) level.

Question 

A 23-year-old man is admitted after experiencing a seizure. A witness to the seizure described how the patient fell to the ground with rigid extension of the arms and legs that was then followed by a period of jerking movements and loss of consciousness. This best describes which type of seizure?

Petit mal.

Myoclonic.

Tonic-clonic (grand mal).

Complex partial.

Question 

A 54-year-old man is being evaluated for pain related to acute bacterial prostatitis. In considering the diagnosis and management, the NP realizes that:

Gram-positive organisms are the most common cause of infection.

Length of antibiotic therapy is usually 1 week.

Perineal pain with defecation is a common complaint.

Cephalosporins are first-line therapy.

Question 

A 25-year-old landscaper with a 6 cm abscess on the left upper arm presents one week post incision and drainage for a follow-up appointment. The area is much improved, but has some residual erythema and discomfort. The nurse practitioner correctly interprets that:

The healing process is proceeding normally and he should continue to monitor progress.

The presence of discomfort one week after incision and drainage suggests residual infection and antibiotics should be started.

The wound should be packed with iodoform gauze and irrigated with normal saline twice daily.

The patient should be referred for a surgical evaluation.

Question 

The Privacy Rule established by the U.S. Department of Health and Human Services in 1996 implemented the requirements for:

Electronic medical records at every healthcare institution.

Health Insurance Portability and Accountability Act (HIPAA).

Expanding Medicare to those <65 years of age who meet certain criteria.

Mandatory psychological screening for all Federal employees.

Question 

In the patient diagnosed with infectious mononucleosis, the use of amoxicillin should be avoided due to the risk of:

Resistance development.

Rash.

Stevens-Johnson syndrome.

Toxic epidermal necrolysis.

Question 

When initiating therapy with spironolactone for a patient who is also on an ACE inhibitor, you advise the patient to return in 5 days to check which of the following laboratory parameters?

Sodium

Calcium

Potassium

Chloride

Question 

A 27-year-old male is rushed to the emergency department after experiencing a series of seizures. He has no history of neurologic disorder. His girlfriend states that he was taking multiple medications for mood disorder but recently stopped taking one of them. The most likely medication the patient discontinued is a:

Selective serotonin reuptake inhibitor (SSRI).

Serotonin-norepinephrine reuptake inhibitor (SNRI).

Benzodiazepine.

Second-generation tricyclic antidepressant.

Question 

When considering an end-of-life decision for a terminally ill person with Alzheimer-type dementia who can no longer communicate, all of the following can be used as an advanced directive except:

Testimony from a family member or close friend

A living will.

A “do not resuscitate” order.

A durable power of attorney for healthcare.

Question 

Which of the following types of clinical studies represents the lowest level of evidence used to guide medical decisions?

Clinical experience of renowned expert in field.

Non-randomized controlled study.

Observational study.

Case-control study.

Question 

Which of the following electrocardiogram (ECG) changes do you expect to find in a person with myocardial ischemia?calcium

Pathologic Q wave

Tall R wave

T wave inversion

ST segment elevation

Question 

You see a 62-year-old woman experiencing a painful episode of acute gouty arthritis. All of the following are appropriate treatment choices to relieve her condition except:

Naproxen sodium (Aleve ® ).

Intraarticular corticosteroid injection.

Allopurinol (Aloprim ® ).

Colchicine (Colcrys ® ).

Question 

As part of the Deficit Reduction Act of 2005, Medicare will incorporate quality payment adjustments for certain hospital-acquired conditions, including all of the following except:

Catheter-associated urinary tract infection.

Myocardial ischemia.

Pressure ulcers.

Deep vein thrombosis.

Question 

In which of the following situations is parental consent usually needed prior to treatment?

A 15-year-old requesting information about contraception.

A 17-year-old who wants help with anxiety.

A 16-year-old requesting treatment for acne vulgaris.

An 18-year-old who requires treatment for depression

Question 

A 17-year-old male with intermittent asthma presents for routine follow-up. He explains that he experiences asthma symptoms once or twice each week, usually during physical activity. Symptoms are promptly relieved each time with albuterol per MDI with spacer. He does not report any nighttime awakenings due to his asthma. His ACT score is 24 and vital signs are all within normal limits. According to the NAEPP EPR-3, what is the next step in the management of his asthma?

Continued use of SABA on an as needed basis.

Initiation of a long-acting beta-agonist.

Initiation of low-dose inhaled corticosteroids.

Restrict physical activity.

Question 

A 41-year-old woman is diagnosed with hypertension that requires medication. She is otherwise healthy but currently taking drospirenone/ethinyl estradiol (Yasmin ® ) for birth control. Which of the following antihypertensive medications would be least preferred for this patient?

Beta-adrenergic antagonist

Angiotensin-converting enzyme inhibitor

Calcium channel blocker

Angiotensin receptor blocker

Question 

A 71-year-old male presents to the emergency department. He is complaining of severe abdominal pain in the right lower quadrant. Diagnostic findings to support a ruptured appendix would include all of the following except:

Leukopenia.

Fever >102ºF (>38.9ºC).

Symptoms lasting more than 48 hours.

Absence of bowel sounds.

Question 

A 57-year-old woman presents with a 6-month history of numbness of the fingertips and oral irritation. Physical examination reveals pale conjunctiva and a heart murmur. She is otherwise healthy, exercises regularly, has a BMI of 22 kg/m 2 , and she states that she is a vegan. Hemogram results are as follows: -Hg=8.2 g/dL (12–16 g/dL) -Hct=23% (36%–42%) -MCV=135 fL (80–96 fL) -Red blood cell distribution width (RDW)=17% (11%–15%) These findings are most consistent with:

Hemolysis.

Vitamin B12 deficiency.

Iron deficiency.

Anemia of chronic disease.

Question 

Which of the following statements is false regarding end-of-life decision-making for a patient that is hopelessly and terminally ill?

A durable power of attorney for healthcare can be used to authorize another person to make healthcare decisions.

A videotaped or audiotaped discussion with the patient can include advanced directives.

Advanced directives are legally binding and recognized in all 50 states.

Living wills and do not resuscitate orders are examples of advanced directives.

Question 

You see a 24-year-old woman of Asian ancestry who is being evaluated for a neurologic disorder. A thorough laboratory analysis is conducted and the hemogram results are as follows: -Hemoglobin (Hg)=10.4 g/dL -Hematocrit (Hct)=32% -Mean cell volume (MCV)=71 fL -Red blood cell distribution width (RDW)=13% -Red blood cells (RBC)=5.5 million The most likely condition associated with these hemogram results is:

Acute blood loss.

Beta thalassemia minor.

Iron deficiency anemia.

Cooley’s anemia.

Question 

The nurse practitioner is evaluating a 19-year-old male suffering from a severe headache. He has a history of headaches that tend to occur in clusters over a few days. He has unsuccessfully tried several therapeutic modalities. A positive response to which of the following interventions supports the diagnosis of cluster headache?

Oxygen therapy.

Beta-adrenergic blockade.

Tricyclic antidepressant.

Dietary reduction of amines.

Question 

A 68-year-old woman who resides in a long-term care facility is being treated for a urinary tract infection. Her medical history includes hypertension, peripheral artery disease, and a 35 pack-year smoking history. During the evaluation, she brings to your attention a painless, pearly nodule on the upper lip. This clinical presentation most likely represents a(n):

Squamous cell carcinoma

Basal cell carcinoma

Actinic keratosis

Molluscum contagiosum

Question 

A 23-year-old male patient is evaluated in the early morning hours for nausea with vomiting. He had eaten some food from the fridge the night before that “was probably too old.” The NP suspects acute gastroenteritis and would expect the course of the disease over the next 48–72 hours to involve:

Continued clinical evidence of dehydration, even with proper fluid uptake

Resolution of vomiting but onset of diarrhea

Continued episodes of vomiting but have improved appetite

Complete resolution of symptoms and be feeling well

Question 

A 47-year-old man is being evaluated for severe eye pain and sudden onset of reduced visual acuity. The NP suspects acute angle-closure glaucoma. Which of the following is most likely to be found on funduscopic examination to support this diagnosis?

Hemorrhagic lesions.

Low intraocular pressure.

Arteriovenous nicking.

A deeply-cupped optic disc.

Question 

A 73-year-old man is being evaluated following an episode of dizziness leading to a fall that did not cause any injury. Cardiac and neurologic examination did not reveal the cause of dizziness. In preparation for discharge planning, the patient undergoes a formal balance assessment using which of the following tests?

Braden scale.

Tinetti assessment.

McMurray test .

Wintrobe criteria.

Question 

All of the following are required Medicare terms and conditions for paying NP services except:

The services are within the NP’s scope of practice as defined by state law.

The services performed are those for which a physician would be able to bill Medicare.

The services are performed in collaboration with a physician.

Separate charges are billed for NP services and facility charges.

Question 

A 21-year-old college student presents with new-onset pain and swelling in the feet and ankles as well as conjunctivitis, oral lesions, and dysuria. She reports that she has had multiple sexual partners and does not use any form of protection. The most important test result to obtain is:

Rheumatoid factor

Sedimentation rate

Urethral cultures

Question 

A 17-year-old male is admitted after experiencing sudden, severe pain in the scrotum during the night. Physical examination reveals swelling of the scrotum and the loss of the cremasteric reflex. His heart rate is 110 bpm, blood pressure 150/80 mm Hg, and temperature 99.8ºF (37.7ºC). This most likely represents:

Testicular neoplasia

Acute epididymitis

Incarcerated hernia

Testicular torsion

Question 

You see a 48-year-old man with a chief complaint of pain during urination. Imaging reveals the presence of a kidney stone in the ureter. You consider all of the following management options except:

Increased fluid uptake with water or citrus drinks.

Alpha blocker use.

Thiazide diuretic use.

Analgesia use.

Question 

According to the Consolidated Omnibus Reconciliation Act (COBRA), the spouse of an eligible employee will be eligible for COBRA coverage in all of the following circumstances except when the eligible employee:

Divorces the spouse.

Becomes eligible for Medicare.

Has his or her hours reduced.

Is terminated for misconduct.

Question 

You see a 14-year-old soccer player with an ankle sprain. Physical examination reveals complete ankle instability, significant swelling, and moderate to severe ecchymosis. You would grade this sprain as:

Grade I.

Grade II.

Grade III.

Grade IV

Question 

A 28-year-old male with asthma presents with a chief complaint that he has to use his rescue inhaler multiple times each day. An evaluation reveals that he has moderate-to-severe asthma and requires Step 4 treatment. An appropriate regimen would include:

Low-dose inhaled corticosteroid (ICS)

Medium-dose ICS

Medium-dose ICS plus a long-acting beta₂-agonist (LABA)

High-dose ICS plus LABA plus omalizumab (Xolair ® )

Question 

A 19-year-old is diagnosed with meningococcal meningitis. In order to prevent secondary infections among adults who had close contact with the patient, all of the following are recommended except:

Administer appropriate meningococcal vaccine if previously unvaccinated.

Prophylaxis with acyclovir (Zovirax ® ).

Prophylaxis with ciprofloxacin (Cipro ® ).

Prophylaxis with Ceftriaxone (Rocephin ® ).

Question 

You see a 26-year-old man with moderate persistent asthma who is experiencing a severe exacerbation. Which of the following would you most likely expect when evaluating this patient?

Crackles.

Inspiratory wheeze.

Expiratory wheeze.

Bradypnea.

Question 

A 62-year-old woman presents in the emergency department complaining of severe toe pain that originated overnight. She has a history of renal disease and is currently taking a thiazide diuretic. Her BMI is 37 kg/m 2 . In considering a diagnosis of acute gouty arthritis, the nurse practitioner knows that the best diagnostic indicator is:

Serum uric acid.

Joint X-ray.

Erythrocyte sedimentation rate (ESR).

Analysis of joint aspirate for urate crystals.

Question 

Which of the following is the most appropriate antibiotic for a 57-year-old man with acute bacterial rhinosinusitis and who has type 2 diabetes mellitus, COPD and a beta-lactam allergy?

Cephalexin (Keflex ® ).

Amoxicillin-clavulanate (Augmentin ® ).

Trimethoprim with or without sulfamethoxazole (Primsol ® , Bactrim ® ).

Levofloxacin (Levaquin ® )

Question 

While taking the history on a 61-year-old female patient, the nurse practitioner learns that she has a 15-year history of poorly-controlled hypertension. The patient admits that another provider had told her that the high blood pressure had affected her eyes. As a result, the nurse practitioner expects that funduscopic examination will likely reveal:

General vessel narrowing

Optic disc atrophy

Macular degeneration

Proliferative retinopathy

Question 

A 28-year-old woman presents who complains of tugging chest pain unrelated to physical activity. She is generally in good health, a non-smoker, has a BMI of 23 kg/m², and denies dyspnea or dizziness. Physical examination reveals a grade 2/6 late systolic murmur that follows a midsystolic click at the 5th intercostal space, mid-clavicular line. An echocardiogram fails to reveal mitral valve tissue redundancy. This clinical presentation is most consistent with:

Silent myocardial infarction.

Aortic stenosis.

Mitral valve prolapse.

Atrial septal defect.

Question 

A physician employs an NP to provide services at a satellite acute care clinic. If the physician is never present at the clinic, which of the following statements is true?

“Incident to” billing is applicable for the services provided by the NP.

The NP services should be billed under the NP’s provider number.

Medicare will pay 100% of the physician rate for the NP services.

The physician must cosign any documentation of services in order to bill Medicare under the physician’s provider number.

Question 

Which of the following describes the ethical principle of beneficence?

The obligation of the healthcare provider to help people in need.

The duty of healthcare provider to do no harm.

The responsibility of the healthcare provider to treat all in the same fair manner.

The right of the competent person to choose a personal plan of life and action.

Is this your assignment or some part of it?
We can do it for you!
Click below to Order
ORDER NOW